Derivative of sin(t/√(t+1)): Calculating with Chain Rule

  • Thread starter Thread starter Youngster
  • Start date Start date
  • Tags Tags
    Derivative
Click For Summary
The discussion focuses on finding the derivative of the function q = sin(t/√(t+1)) using the chain rule. The initial derivative is expressed as cos(t/√(t+1)) multiplied by a complex fraction involving t and (t+1). Participants clarify that simplifying the derivative of (t(t+1))^(-1/2) is essential for arriving at the correct answer. The final expression for the derivative is confirmed as cos(t/√(t+1)) * (t+2)/(2(t+1)^(3/2)). The conversation highlights the importance of simplification and understanding exponent rules in calculus.
Youngster
Messages
37
Reaction score
0

Homework Statement



Find the derivative of the function: q = sin (\frac{t}{\sqrt{t+1}})

Answer: cos (\frac{t}{\sqrt{t+1}}) (\frac{t+2}{2(t+1)^{\frac{3}{2}}})

Homework Equations



Chain Rule
\frac{dq}{dt} sin x = cos x

The Attempt at a Solution



\frac{dq}{dt} = cos (\frac{t}{\sqrt{t+1}}) \frac{dq}{dt} (t(t+1))^{-\frac{1}{2}} = cos (\frac{t}{\sqrt{t+1}}) (t(-\frac{1}{2}(t+1)^{-\frac{3}{2}} + 1(t+1)^{-\frac{1}{2}})

So that's as far as I've gotten with this problem. I unfortunately don't know how to continue with it though. Does simplifying the derivative of (t(t+1))^{-\frac{1}{2}} lead me to the answer provided? Or did I derive something wrong?
 
Physics news on Phys.org
Yes, you're right simplification will lead you to the right answer.

so you have

-t/(2(t+1)^(3/2)) + 1/(t+1)^(1/2)

so the common denominator is 2(t+1)^(3/2) so multiply the top and bottom of the second expression by 2(t+1)

so you have

-t/(2(t+1)^(3/2)) + 2(t+1)/(2(t+1)^(3/2))

add them now

2t+2-t/(2(t+1)^(3/2))

t+2/(2(t+1)^(3/2))
 
Ah thank you! I've figured it out now and learned a new thing about exponents.
 
Question: A clock's minute hand has length 4 and its hour hand has length 3. What is the distance between the tips at the moment when it is increasing most rapidly?(Putnam Exam Question) Answer: Making assumption that both the hands moves at constant angular velocities, the answer is ## \sqrt{7} .## But don't you think this assumption is somewhat doubtful and wrong?

Similar threads

  • · Replies 105 ·
4
Replies
105
Views
6K
  • · Replies 3 ·
Replies
3
Views
2K
  • · Replies 1 ·
Replies
1
Views
2K
Replies
8
Views
1K
  • · Replies 6 ·
Replies
6
Views
1K
Replies
1
Views
2K
Replies
6
Views
2K
  • · Replies 3 ·
Replies
3
Views
2K
Replies
5
Views
2K
  • · Replies 9 ·
Replies
9
Views
1K